Search found 95 matches


Thanks buddy. I still have a year to apply. I am about to begin on essays. What about you ? Are you reappearing for the GMAT ?

by gmat620

Sun Dec 27, 2009 3:05 am
Forum: I just Beat The GMAT!
Topic: An even 700, Please throw your thoughts.
Replies: 2
Views: 1973

An even 700, Please throw your thoughts.

Hello friends, I took the GMAT second time and scored 700 (V37, Q49). I am confused now. Its 700 so its difficult to decide. What do you people think ? Should I retake ? I am asking this funny question because I have seen many people in this forum planning to retake at 690. Also, my low Verbal is du...

by gmat620

Fri Dec 11, 2009 11:17 am
Forum: I just Beat The GMAT!
Topic: An even 700, Please throw your thoughts.
Replies: 2
Views: 1973

Congrats for rocking score !! All the best for application procedure .

Could u please tell me the level of difficulty for quant in real exam as compared to GPrep and MGMAT ?

by gmat620

Thu Dec 03, 2009 2:03 am
Forum: I just Beat The GMAT!
Topic: 700 - I wanna keep trying
Replies: 3
Views: 1849

A is wrong because any action which is expected to reduce the aggregate well being of ppl will automatically fall into category of wrong action by definition. Wrong Action definition: An action is morally wrong if and only if it would be reasonably expected to reduce the aggregate wellbeing of the p...

by gmat620

Wed Dec 02, 2009 8:50 pm
Forum: Critical Reasoning
Topic: LSAT CR
Replies: 7
Views: 2743

Congrats for your great score bro !! Good luck for application.

by gmat620

Tue Dec 01, 2009 3:53 pm
Forum: I just Beat The GMAT!
Topic: Finally a 710 (V 39,Q 49) 3rd trial , my lessons learned :)
Replies: 4
Views: 2300

Dount regarding past subjunctive

I have doubt lurking in my mind since I saw a question in Gprep which has subjunctive in past form.The present tense of the sentence is just like this: The federal rules are established to ensure that patients be warned before check up and that independent doctor evaluate the case. Can anybody pleas...

by gmat620

Tue Dec 01, 2009 12:33 pm
Forum: Sentence Correction
Topic: Dount regarding past subjunctive
Replies: 1
Views: 1023

Many thanks to you Stacey. As always I would listen to your advice Stacey but I have limited time and tight schedule so I was convincing myself that GMAT wouldn't throw that hard questions. I heard that real GMAT is far tougher than GMAT PREP and MGMAT CAT .Is that true ? Please enlighten me. One mo...

by gmat620

Wed Nov 25, 2009 2:17 pm
Forum: Critical Reasoning
Topic: 700-800 level CRs
Replies: 4
Views: 2787

I am in the same boat friend...I am suffering in CAT's only because of this thing called CR. I did OG and don't find many hard questions but on MGMAT CAT, I felt like dying. I couldn't decipher the correct one even after 2 minutes. I have the test next week so I am really tensed but I hope GMAT woul...

by gmat620

Mon Nov 23, 2009 4:56 pm
Forum: Critical Reasoning
Topic: 700-800 level CRs
Replies: 4
Views: 2787

Strong concept tested: Please help me out

Is M < 0 ?

1. - M = | - M |
2. M^2 = 9

Hello Friends, I think mod is always positive, please help me out on this one...This question isn't tough but concept is required. I am little bit confused. Please help me. Thanks in advance.

by gmat620

Sat Nov 21, 2009 3:29 am
Forum: Data Sufficiency
Topic: Strong concept tested: Please help me out
Replies: 3
Views: 1094

good explanation by palvarez !!

by gmat620

Wed Nov 18, 2009 9:32 pm
Forum: Problem Solving
Topic: probability problem with condition
Replies: 4
Views: 1588

Tricky one... IMO E Option E.....yes the prices are low in town A and that makes them more aware about international sports because they read more (they have cheap access to media so they are better informed....this strengthens the argument that they are well aware ). Option C weakens the argument b...

by gmat620

Tue Nov 17, 2009 3:00 pm
Forum: Critical Reasoning
Topic: Sports magazine.
Replies: 8
Views: 2374

IMO E 1> sqrt (x) > y x > y^2 but what if we have a decimal value, for example x = 0.1, y= 0.2 y^2 = .04 which is less than x (=.10) so we can't be sure whether x ^3 >y 2> x > y if x = 5, y =4 then yes we have x^3 > y but x = 0.2, y = 0.3 , then NO because (x^3 =).008 < 0.3 (y) so insufficient.. now...

by gmat620

Sat Nov 14, 2009 11:02 am
Forum: Data Sufficiency
Topic: inequal...ds roots
Replies: 6
Views: 1864

But why not A. May be increase in road fares caused ppl to switch to public transport. How to eliminate A.

by gmat620

Fri Nov 13, 2009 8:32 am
Forum: Critical Reasoning
Topic: Easy but confusing one from OG-11: please help
Replies: 5
Views: 2102

Easy but confusing one from OG-11: please help

Since the mayor’s publicity campaign for Greenville’s bus service began six months ago, morning automobile traffic into the midtown area of the city has decreased seven percent. During the same period, there has been an equivalent rise in the number of persons riding buses into the midtown area....

by gmat620

Fri Nov 13, 2009 2:56 am
Forum: Critical Reasoning
Topic: Easy but confusing one from OG-11: please help
Replies: 5
Views: 2102